Lesson: Assumption Questions

Comment on Assumption Questions

gmat-admin's picture

Question link: https://gmatclub.com/forum/commentator-the-theory-of-trade-retaliation-s...

We're told that, if Country X closes some of its markets to Country Y, then Country Y should close some of it's markets to Country X.

CONCLUSION: If every country acted this way, no country would trade with any other.

Let's use the NEGATION technique:

(A) SOME countries actually act according to the theory of trade retaliation.
Does this destroy the conclusion that "If every country acted this way, no country would trade with any other"?
Not really.
If 1 or 2 countries act this way, but no other countries do, then most countries will still trade with each other.

(D) For any two countries, NEITHER country has some market closed to the other.
Does this destroy the conclusion that "If every country acted this way, no country would trade with any other"?
YES!
If no countries are closing some of their markets to other countries, then trade between countries will continue as usual.

Answer: D

Does that help?

Cheers,
Brent

It is english language possible to write? Sorry for my proor eng
gmat-admin's picture

Sorry, but I don't understand your question. Can you rephrase it?

Hi Brent,

At 05:41, could you analyze the other two assumption too please? Especially the last one, I´m wondering why this is not the correct answer? Thank you x
gmat-admin's picture

Those other two assumptions are also crucial to the argument.
In fact, arguments can have lots of necessary assumptions. This is why the question stem typically asks "Which of the following is AN assumption on which the argument depends?" (as opposed to "Which of the following is THE assumption on which the argument depends?"

Assumption 1: Nothing stops the championship.
If we negate this, we get: The championship does NOT occur.
This clearly destroys the conclusion, since Juan can't win the championship if there is no championship.

Assumption 2: Juan is eligible to play in the championship.
If we negate this, we get: Juan is NOT eligible to play in the championship
This clearly destroys the conclusion, since Juan can't win the championship if he in NOT eligible to play in championship.

Does that help?

Cheers,
Brent

Hi Brent,

I don't understand why the answer choice A is the correct one and C is the wrong one?

What type of question it is? I read comments on GMATCLUB and I still don't understand why if we know that sales went up 10% for me it is already implied that we are talking about total sales(on what other services gasoline stations may generate revenue).

https://gmatclub.com/forum/last-august-the-xt-chain-of-gasoline-stations-had-a-temporary-sales-160887.html

Thank you in advance,
gmat-admin's picture

Link: https://gmatclub.com/forum/last-august-the-xt-chain-of-gasoline-stations...

The question stem: In evaluating the argument, it would be most helpful to answer which of the following?
This is an Evaluate the Conclusion question. More here: https://www.gmatprepnow.com/module/gmat-critical-reasoning/video/1144

The argument here is that the sales promotion (i.e., buy 10+ gallons and get free car wash) CAUSED the 10% increase in sales.
However, if it were the case that ALL gas stations (XT gas stations and non-XT gas stations) in the area saw a 10% increase, then that would seriously undermine the conclusion that the sales promotion CAUSED the 10% increase in sales.

(A) In the areas in which XT’s gasoline stations operate, how did total combined gasoline sales for all gasoline stations last August compare with sales for the previous August?

Knowing the answer to this question would help determine whether the sales promotion caused the 10% increase in sales.

Cheers,
Brent

Hi Brent,

Refer to the lesson

6.Apply Negation Technique:
"Negate each answer choice and insert it into the argument.
The negated answer choice that destroys the argument is the correct answer"

As above instruction, doesn't it mean that "weaken and an assumption" question types are similar rather than "strengthen"? since we need to find the answer choice that against the conclusion of the argument.

gmat-admin's picture

For assumption questions, we aren't trying to identify "the answer choice that [goes] against the conclusion of the argument. We're trying to find an answer choice that is 100% necessary in order to draw the conclusion.

It's only when we apply the Negation Technique that we are looking for the answer choice that destroys the conclusion.

In this sense, Assumption questions are similar to Strengthen the Argument questions in that the correct answer choice improves the validity of the conclusion.

The only difference is that, in Assumption questions, the missing assumption is CRITICAL to the conclusion, and in Strengthen the Argument questions, the missing premise HELPS strengthen the conclusion

Does that help?

Cheers,
Brent

Thanks for your response, I might misunderstand some points in here.
For weaken/strengthen question, I can pick the correct answer around 80% but only 10 % correct in assumption.
gmat-admin's picture

It sounds like you might have a misconception about what Assumption questions are asking you to do.
I suggest that you start by re-watching the above video.

Then I suggest that you try answering the following video Assumption questions:
- https://www.gmatprepnow.com/module/gmat-critical-reasoning/video/1191
- https://www.gmatprepnow.com/module/gmat-critical-reasoning/video/1205
Note: Be sure to watch the video solutions.

Beyond that, I suggest that you start with some of the easier Assumption questions to help build your skills:
- https://gmatclub.com/forum/in-recent-years-many-cabinetmakers-have-been-...
- https://gmatclub.com/forum/because-no-employee-wants-to-be-associated-wi...
- https://gmatclub.com/forum/installing-scrubbers-in-smokestacks-and-switc...
- https://gmatclub.com/forum/although-there-is-no-record-of-poet-edmund-sp...
- https://gmatclub.com/forum/economist-the-most-economically-efficient-way...
- https://gmatclub.com/forum/infotek-a-computer-manufacturer-in-katrovia-h...

I hope that helps.

Cheers,
Brent

Hi Brent,

https://gmatclub.com/forum/sviatovin-is-a-medieval-moringian-text-whose-author-and-exact-date-of-136788.html

By process of elimination, I deducted that D is the correct choice, but I spent a lot of time on it.

I was confused by the meaning of "family composition". Does it include who died and when, or it just includes information about the relationships between family members? If the former, then the answer choice makes sense if the latter then if the uncle was mistaken for the father does not change the fact that he died in 1167, which is all I need to make sure when the Moringian was written.

What do you think?
gmat-admin's picture

Question link: https://gmatclub.com/forum/sviatovin-is-a-medieval-moringian-text-whose-...

I believe that the "family composition" describes all of the relationships, children, marriages, etc. A family tree.
It's possible that some family trees note date of deaths, but that wouldn't be relevant here anyway, since the father was said to be alive when the Moringian was written. That is, at the time the Moringian was written, the father's death was still in the future.

Ok, but if knowledge about family composition is not relevant, why then D the correct choice? Simply because all other choices are wrong?
gmat-admin's picture

But C is the correct choice.

C) the diagram accurately represents the composition of Sviatov's family at the time Sviatovin was written.
Let's apply the Negation Technique: If the diagram is wrong and the person assumed to be the father is not the father, then that kills the conclusion.

Sorry, my bad,

Thank you

Hi Brent,

Could you explain how would you explain to yourself that C must be a correct answer and not E?

Kitchen magazine plans to license the use of its name by a line of cookware. For a magazine, licensing the use of its name for products involves some danger, since if the products disappoint consumers, the magazine's reputation suffers, with consequent reduction in circulation and advertising. However, experts have evaluated the cookware and found it superior to all other cookware advertised in Kitchen. Therefore, Kitchen can collect its licensing fees without endangering its other revenues.

The argument above assumes which of the following?


(A) No other line of cookware is superior to that which will carry the Kitchen name.

(B) Kitchen will not license the use of its name for any products other than the line of cookware.

(C) Makers of cookware will not find Kitchen a less attractive advertising vehicle because the magazine's name is associated with a competing product.

(D) Consumers who are not regular readers of Kitchen magazine will be attracted to the cookware by the Kitchen name.

(E) Kitchen is one of the most prestigious cooking-related magazines.

Thank you in advance,
gmat-admin's picture

Question link: https://gmatclub.com/forum/kitchen-magazine-plans-to-license-the-use-of-...

For this question we must be super careful about identifying the conclusion. The conclusion does NOT say that Kitchen magazine will increase profits by licensing its name to a line of cookware. Also, the conclusion does NOT say that Kitchen magazine will increase its revenue.

The conclusion says "Kitchen can collect its licensing fees without endangering its OTHER revenues"

Let's start by negating answer choice C to get: Makers of cookware WILL find Kitchen a less attractive advertising vehicle because the magazine's name is associated with a competing product.
So by licensing its name to a line of cookware, Kitchen magazine will become a less attractive advertising vehicle.
This negated answer choice, seriously harms the conclusion that "Kitchen can collect its licensing fees without endangering its OTHER revenues"

Now let's negate answer choice E to get: Kitchen is NOT one of the most prestigious cooking-related magazines.
This negated answer choice has no effect on the conclusion that "Kitchen can collect its licensing fees without endangering its OTHER revenues"

Cheers,
Brent

Hi Brent,

This was a definite misunderstanding from my side of the conclusion,

Thank you for your help,

Now I understand,

Hi Brent,

Could you please break down the argument and assumtion for this question:

The city of Workney, in raising bus fares from $1.00 to $1.25, proposed that 18 fare token be sold for $20.00 to alleviate the extra burden of the fare increase on the city's low-income residents. Critics suggested alternatively that 9 fare tokens be sold for $10.00, because a $20.00 outlay would be prohibitive for low-income riders.

The alternative proposal depends on which of the following assumptions?


A. Low-income residents of Workney will continue to ride the buses in the same numbers despite the fare increase

B. Low-income riders would be more likely to take advantage of the savings afforded by the 9-token offer than would other riders.

C. The outlay of $10.00 for the purchase of 9 fare tokens would not be prohibitive for low-income bus riders

D. The proposed fare increase is needed for the purchase of new buses for the city's bus system

E. Buses provide the only means of public transportation in the city of Workney.


For me when you have to pay $20 for 18 fare token is exactly the same rate as paying $10 for 9 fare token. So what is the difference? And what is the point of the alternative solution?

Thank you in advance,
gmat-admin's picture

Question link: https://gmatclub.com/forum/the-city-of-workney-in-raising-bus-fares-from...

Hi Kirill. You're correct to say that paying $20 for 18 tokens is equivalent to paying $10 for 9 tokens.
However, the passage suggests that many of Workney's poor people might not have $20 in their pocket at any given time.
So, those poor people would not be able to take advantage of the discounted $20 for 18 tokens.
For this reason, the critics suggested it may be better for the poor people if the city sold 9 tokens for $10.

CONCLUSION: Selling 9 tokens for $10 will alleviate the extra burden on poor people.

When we apply the Negation Strategy to answer choice C we get:
The outlay of $10.00 for the purchase of 9 fare tokens WOULD BE prohibitive for low-income bus riders.

This NEGATED assumption destroys the conclusion that "selling 9 tokens for $10 will alleviate the extra burden on poor people."
If the $10 cost would be prohibitive for low-income bus riders, then the $10 price tag of the discounted tokens would not help poor people at all.

Does that help?

Hi Brent,

That is very clear,

Thank you for such a nice explanation,

I understand now what was the point.

Hi Brent,

https://gmatclub.com/forum/infotek-a-computer-manufacturer-in-katrovia-has-just-introduced-a-220526.html

Could you please explain this question? I narrowed it down to B & C and had a tough time choosing between the two. I get why C is correct but for B -

If we know:
1) new PC is cheaper 2) Few people buy PC, regardless of price and with the new assumption 3) People care about the improved model => Conclusion: new model won't change the PC usage or Demand for PC.

That made sense to me! I would really appreciate some further help on this question as other explanations state that B is irrelevant.

Thanks!
gmat-admin's picture

Question link: https://gmatclub.com/forum/infotek-a-computer-manufacturer-in-katrovia-h...

The conclusion: Introducing the new model is unlikely to INCREASE THE NUMBER of computers in Katrovian homes.
There are two ways to increase the number of computers in Katrovian homes:
1) people who don't have a computer buy a computer
2) people who already have a computer buy another computer

Let's examine B and C

(B) The main reason cited by consumers in Katrovia for REPLACING a personal computer is the desire to have an improved model.
There are two problems with this answer choice:

First, we don't know whether Infotek's new computers are better than existing computers, the same as existing computers, or worse than existing computers.
As such, we can't really determine the validity about the choice B.

More importantly, answer choice B involves REPLACING existing computers with Infotek computers. This means the NUMBER of computers in Katrovian homes would not change.

For these reasons, answer choice B is no good.

(C) Katrovians in households that already have computers are unlikely to purchase the new Infotek model as an ADDITIONAL computer for home use.
let's see what happens if we NEGATE answer choice C.
We get: Katrovians in households that already have computers are LIKELY to purchase the new Infotek model as an ADDITIONAL computer for home use.
This negated assumption DESTROYS the conclusion that introducing the new model is unlikely to INCREASE THE NUMBER of computers in Katrovian homes.

As such the correct answer is C

Does that help?

Hi Brent,

I have a question on negation, sorry if you have covered this already.

I am confused with how some words will be negated.
Such as- will 'some'become - none or many.

Here is what I read on GmatClub in this article - https://gmatclub.com/forum/article-what-and-how-to-negate-6-exercise-questions-138510.html

Could you help share the right negation for the most common words.

Thank you.
gmat-admin's picture

When negating and answer choice, I typically start by adding "IT IS NOT THE CASE THAT" to the beginning of the answer choice. Doing so is typically all you need to understand what the negated answer choices telling us.

That said, here are a few more negations.

SOME means "one or more."
So, the negation of SOME is "none"

The negation is NONE is "some"

The negation of MANY is "not many" (notice that "not many" doesn't really tell us anything)

Cheers, Brent

Hi Brent. Good Evening.
In 'Reinforcement Activities' question - 'https://gmatclub.com/forum/although-the-school-would-receive-financial-benefits-if-it-had-soft-dr-202757.html,
I am confused between option 'A' & 'D'. Can you please explain how 'D' doesn't fit in as a solution?
gmat-admin's picture

Question link: https://gmatclub.com/forum/although-the-school-would-receive-financial-b...

Conclusion: Adding soft drink machines to the cafeteria is not in the students' interest
IMPLIED conclusion: Adding soft drink machines to the cafeteria would negatively affect students' health.

(A) If the soft drink vending machines were placed in the cafeteria, students would consume more soft drinks as a result.
If we NEGATE this answer choice, we get: If the soft drink vending machines were placed in the cafeteria, students would NOT consume more soft drinks as a result.
This negated answer choice destroys the conclusion.
So, we'll keep it.

(D) Students will not simply bring soft drinks from home if the soft drink vending machines are not placed in the cafeteria.
If we NEGATE this answer choice, we get: Students WILL bring soft drinks from home if the soft drink vending machines are not placed in the cafeteria.
This tells us that NOT adding soft drink machines to the cafeteria will still negatively affect students' health.
This, however, does not destroy the conclusion, since the conclusion has nothing to do with what happens if soft drink machines are NOT added to the cafeteria.

Does that help?

Hi Brent. Good Evening.
In 'Reinforcement Activities' question -
https://gmatclub.com/forum/the-technological-conservatism-of-bicycle-manufacturers-is-a-reflectio-117503.html, how is 'C' a solution? Please explain.
gmat-admin's picture

Question link: https://gmatclub.com/forum/the-technological-conservatism-of-bicycle-man...

Implied conclusion: Since racers are the only ones interested in "better" bicycles, manufacturers should focus solely on innovation that will help racers compete.

(C) Bicycle racers do not generate a strong demand for innovations that fall outside what is officially recognized as standard for purpose of competition.

When we negate this we get: (C) Bicycle racers DO generate a strong demand for innovations that fall outside what is officially recognized as standard for purpose of competition.
This negated premise destroys the argument, since it tells us there's also a demand for innovation that's not related to competition.

As such, C is the best answer.

Hi Brent. Good Evening.
In 'Reinforcement Activities' question -
https://gmatclub.com/forum/birds-have-been-said-to-be-descended-from-certain-birdlike-dinosaur-220394.html, I couldn't understand your solution 'C' on this. Can you please elaborate to explain?
gmat-admin's picture

Question link: https://gmatclub.com/forum/birds-have-been-said-to-be-descended-from-cer...

Argument summary:
It SEEMS like birds descended from bird-like dinosaurs.
However, there are bird fossils that are older than the oldest bird-like dinosaur fossils.
Conclusion: Birds did not descend from bird-like dinosaurs

When we negate answer choice C, we get: There ARE birdlike dinosaur fossils that are older than the bird fossils but have not yet been unearthed.
This negated answer choice exposes a huge assumption in the original argument: just because scientists haven't FOUND bird-like dinosaur fossils that are older than bird fossils, we can't assume that such bird-like dinosaur fossils don't exist.

Here's an analogous argument: Many people believe that the Canadian mint started producing 1-dollar coins BEFORE it started producing 2-dollar coins. The coins in my pocket, however, dispute this, since there are 2-dollar coins in my pocket that are older then the oldest 1-dollar coin in my pocket.

Answer choice C, when negated, destroys the original conclusion by suggesting there ARE bird-like dinosaur fossils that are older than bird fossils.
As such, it's the best answer.

gmat-admin's picture

Question link: https://gmatclub.com/forum/birds-have-been-said-to-be-descended-from-cer...

Argument summary:
It SEEMS like birds descended from bird-like dinosaurs.
However, there are bird fossils that are older than the oldest bird-like dinosaur fossils.
Conclusion: Birds did not descend from bird-like dinosaurs

When we negate answer choice C, we get: There ARE birdlike dinosaur fossils that are older than the bird fossils but have not yet been unearthed.
This negated answer choice exposes a huge assumption in the original argument: just because scientists haven't FOUND bird-like dinosaur fossils that are older than bird fossils, we can't assume that such bird-like dinosaur fossils don't exist.

Here's an analogous argument: Many people believe that the Canadian mint started producing 1-dollar coins BEFORE it started producing 2-dollar coins. The coins in my pocket, however, dispute this, since there are 2-dollar coins in my pocket that are older then the oldest 1-dollar coin in my pocket.

Answer choice C, when negated, destroys the original conclusion by suggesting there ARE bird-like dinosaur fossils that are older than bird fossils.
As such, it's the best answer.

Hi Brent. Good Evening.
In 'Reinforcement Activities' question -
https://gmatclub.com/forum/proposed-new-safety-rules-for-the-beach-city-airport-would-lengthen-201652-40.html, though I chose correct answer 'E' but I was not fully confident about it. Can you please explain the answer to me?
gmat-admin's picture

Question link: https://gmatclub.com/forum/proposed-new-safety-rules-for-the-beach-city-...

Argument summary: New safety rules will result in 10% fewer daily flights.
The city's budget depends on taxes on tourist spending, and most tourists arrive by plane.
Conclusion: The new safety rules will reduce the revenue generated by the city.

Notice that it's assumed that a 10% reduction in flights will cause a reduction in the number of tourists.

Answer choice E exposes that assumption. It says:
(E) The response to the adoption of the new safety rules would not include an increase in the number of passengers per flight.

If we NEGATE this answer choice we get: The response to the adoption of the new safety rules WOULD INCLUDE an increase in the number of passengers PER FLIGHT.
So, even though there will be fewer flights, it might actually be the case that there will be more tourists (due to the decrease number of passengers per flight)
This kills the conclusion.

Hi Brent. Good Evening.
In 'Reinforcement Activities' question -
https://gmatclub.com/forum/a-researcher-discovered-that-people-who-have-low-levels-of-immune-syst-116407-20.html, the answer 'D' is very confusing. Can you please help here?
gmat-admin's picture

Question link: https://gmatclub.com/forum/a-researcher-discovered-that-people-who-have-...

Argument summary:
People with LOW ISA (immune-system activity) have low mental health
People with NORMAL/HIGH ISA have good mental health
Conclusion: The immune system protects against mental illness

Important: Notice that this is a cause and effect argument, and the implied conclusion is a bad immune system CAUSES mental illness.

When we take answer choice D and negate it, we get: (D) Mental illness CAUSES people’s immune-system activity to decrease.
This destroys the implied conclusion by saying the causation works the other way around.
As such, the correct answer is D

Hi Brent,
Can you please explain this question and solution:
https://gmatclub.com/forum/the-reason-much-refrigerated-food-spoils-is-that-it-ends-up-out-of-sig-16046.html

Hi Brent,

I seem to be getting most( if not all I guess) of the argument questions wrong. I've tried the negation technique and have found that there are usually a couple of answer choices which do seem absolutely important to hold the argument true.

How do I improve on this? What other questions should I ask myself when evaluating the answer choices?
gmat-admin's picture

Trying to identify possible assumptions is typically hard, whereas the negation technique is easier to perform (even though it's more time-consuming)

Some students have difficulty with assumption questions primarily because they don't accurately identify the conclusion of the argument.
If the conclusion isn't quite right, then it's going to be extra difficult to identify the correct answer.
So, while identifying the conclusion, be sure to ask yourself "What is this argument trying to convince me of?" Take your time with this step, because it's crucial to solving the question.

Also, don't forget that, in order for an answer choice to be correct, the negated version must DESTROY the argument (i.e., the conclusion no longer follows from the premises). So, if the negated version merely injures/dents the argument, that answer choice isn't correct.

I can't think of anything else beyond the tips noted in the video.

Hi Brent, getting a bit confused with using "it is not the case that" especially when the answer choice already have negative words in it as in option D in question below: Could you help? Thanks Brent

https://gmatclub.com/forum/in-recent-years-many-cabinetmakers-have-been-winning-acclaim-as-artist-69962.html

Great thanks Brent make sense now.

Pages

Office Hours

On December 20, 2023, Brent will stop offering office hours. 

Change Playback Speed

You have the option of watching the videos at various speeds (25% faster, 50% faster, etc). To change the playback speed, click the settings icon on the right side of the video status bar.

Have a question about this video?

Post your question in the Comment section below, and a GMAT expert will answer it as fast as humanly possible.

Free “Question of the Day” emails!